Back + Spinal Cord - MCQ Questions

Réussis tes devoirs et examens dès maintenant avec Quizwiz!

*The answer is D.* The conus medullaris is usually located at the L1 to L2 vertebral level; therefore, any choice that contains that region is the correct answer. L3 to L4 is a common location to perform lumbar puncture, but it is caudal to the apex of the conus medullaris. L3 and L4 are caudal to the conus medullaris. T11 is superior to the conus medullaris.

A 66-year-old woman had been diagnosed with a tumor on her spine. She has started to retain urine and is experiencing rectal incontinence. Both of these symptoms are signs of conus medullaris syndrome. At which of the following vertebral levels is the tumor probably located? A. L3/L4 B. L3 C. L4 D. T12 to L2 E. T11

*The answer is A.* Multifidus is a deep muscle, which attaches from the transverse processes to the spinous processes usually crossing four to six segments. Longissimus, iliocostalis, and spinalis are not deep to semispinalis but are superficial. The rotators typically attach between the spinous processes or lamina of vertebrae and the transverse processes of the vertebra one or two segments below.

Which of the following muscles is most likely located immediately deep to the semispinalis muscles, pass from a lateral point of origin in a superomedial direction to attach to spinous processes, and cross between 2 and 4 vertebrae? A. Multifidus B. Rotatores C. Longissimus D. Iliocostalis E. Spinalis

*The answer is C.* Lumbar puncture is generally performed at the level of L4, L5. The spinal cord ends at the level of L1/L2 in adults and at the level of L2/L3 in newborns.

A 3-day-old girl develops a fever. She is irritable and not feeding. As part of the workup for fever of unknown origin, a lumbar puncture is performed. This puncture must be done below the spinal cord which usually ends at which vertebral level in a patient of this age? A. L1 B. S1 C. L3 D. S3 E. L5

*The answer is D.* The dorsal primary rami of the spinal nerves innervate the deep muscles of the back, including the iliocostalis. The other muscles are the superficial muscles of the back, which are innervated by the ventral primary rami of the spinal nerves.

A 34-year-old woman crashes into a tree during a skiing lesson and is brought to a hospital with multiple injuries that impinge the dorsal primary rami of several spinal nerves. Such lesions could affect which of the following muscles? (A) Rhomboid major (B) Levator scapulae (C) Serratus posterior superior (D) Iliocostalis (E) Latissimus dorsi

*The answer is C.* Meningomyelocele is protrusion of the meninges and spinal cord through the unfused arch of the vertebra. Sufficient amount of folic acid during pregnancy is shown to prevent these kind of neural tube defects. Spina bifida occulta is failure of the vertebral arch to fuse (bony defect only). Meningocele is protrusion of the meninges through the defective vertebral arch. Syringomyelocele is protrusion of the meninges and a pathologic tubular cavity in the spinal cord or brain.

A young toddler presents to her pediatrician with rather new onset of bowel and bladder dysfunction and loss of the lower limb function. Her mother had not taken enough folic acid (to the point of a deficiency) during her pregnancy. On examination, the child has protrusion of the spinal cord and the meninges and is diagnosed with which of the following conditions? (A) Spina bifida occulta (B) Meningocele (C) Meningomyelocele (D) Myeloschisis (E) Syringomyelocele

*The answer is C.* Spina bifida cystica refers to spina bifida with a meningocele or myelomeningocele and is the correct answer. Cranium bifida could present with meningo-cele in the skull, but it would not be located in the lower back. Spina bifida occulta is a defect in the formation of the vertebral arches and does not usually present with meningocele. Hemothorax refers to blood accumulation in the pleural space surrounding the lungs. Caudal regression syndrome presents with loss or deformation of the distal part of the spine and/or spinal cord and is not related to a meningocele or myelomeningocele, in general.

A maternal serum sample with high alpha-fetoprotein alerted the obstetrician to a possible neural tube defect. Ultrasound diagnosis revealed a myelomeningocele protruding from the back of the child. Which of the following is the most likely diagnosis of this congenital anomaly? A. Cranium bifida B. Spina bifida occulta C. Spina bifida cystica D. Hemothorax E. Caudal regression syndrome

*The answer is A.* The muscle that forms the lateral border of the suboccipital triangle is the obliquus capitis superior. This muscle originates from the transverse process of the atlas and inserts untoon the occipital bone between superior and inferior nuchal lines. The muscle that inserts at the transverse process of the atlas is the obliquus capitis inferior which forms the inferior border of the suboccipital triangle. The rectus capitis posterior major inserts on the lateral portion of occipital bone below the inferior nuchal line and the rectus capitis posterior minor inserts on the medial portion of occipital bone below the inferior nuchal line. These muscles form the medial border of the triangle. The rectus capitis posterior minor originates from the posterior tubercle of the atlas.

A radiology report of a cervical spine MRI scan contains the following statement: "A small 1-cm tumor is located within a muscle on the lateral border of the right suboccipital triangle." The muscle to which the radiologist is most likely referring inserts on which of the following bony features? A. Transverse process of atlas B. Lateral portion of occipital bone below inferior nuchal line C. Occipital bone between superior and inferior nuchal lines D. Medial portion of occipital bone below inferior nuchal line E. Posterior tubercle of atlas

*The answer is B.* The levator scapulae arises from the transverse processes of the upper cervical vertebrae and inserts on the medial border of the scapula. The other muscles are attached to the spinous processes of the vertebrae.

After his car was broad-sided by a large truck, a 26-year-old man is brought to the emergency department with multiple fractures of the transverse processes of the cervical and upper thoracic vertebrae. Which of the following muscles might be affected? (A) Trapezius (B) Levator scapulae (C) Rhomboid major (D) Serratus posterior superior (E) Rectus capitis posterior major

*The answer is E.* The posterior (dorsal) primary rami of spinal nerves innervate the embryonic epimere and all the skeletal muscles derived from it. The derivatives of the epimere constitute the deep (intrinsic) muscles of the back (or epaxial muscles), including the splenius capitis and cervicis muscles, suboccipital muscles, transversospinales muscles (including the semispinalis, multifidus, and rotatores muscles), and several other small muscles. Thus, damage to the cervical posterior (dorsal) primary rami, specifically C2-6, would result in paralysis of the splenius capitis muscle.

As a result of multiple vertebral fractures incurred in an automobile crash, an 8-year-old girl suffers a series of torn posterior primary rami of spinal nerves C1-6. Which of the following muscles will be paralyzed as a result? (A) Trapezius (B) Latissimus dorsi (C) Levator scapulae (D) Rhomboid major (E) Splenius capitus

*The answer is E.* The medullary cone (conus medullaris) is the tapered terminal end of the spinal cord, composed of the sacral and coccygeal segments. In adults, the conus medullaris typically lies within the T12-L3 vertebral levels, but it generally ends at approximately L2. Thus, displacement fractures of these vertebrae are likely to affect one or more of the sacral-coccygeal spinal cord segments. In this case, given the statistical range of variation for the medullary cone, the displaced fracture fragment of the L1 vertebral body is more likely to impinge the lower lumbar and upper sacral segments of the medullary cone. S2 is the only choice from this region. When thinking of the spinal cord, remember the "rule of 2," which states the conus medullaris usually ends at the second lumbar vertebra (L2); the subarachnoid space ends at the second sacral vertebra (S2), and the terminal filum (filum terminale), a long connective tissue (pia mater) strand extending from the end of the medullary cone that anchors the inferior aspect of the spinal cord, usually attaches to the second coccygeal vertebra (Co2). Due to the location of the displaced fracture fragment of the L2 vertebral body, the higher sacral segments of the spinal cord related to the medullary cone are more likely to be damaged.

Attempting to do a backflip with his bicycle off a high ramp at the fi nish line of a race, a 24-year-old professional BMX rider fell from a height of 20 ft and attempted to land on his feet. A sagittal CT reveals a burst fracture of the L1 vertebral body with a posterior displaced fracture fragment compressing the medullary cone (conus medullaris). Which of the following spinal cord segments would most likely be impinged by the bone fragment in this injury? (A) T7 (B) T9 (C) T11 (D) L2 (E) S2

*The answer is A.* The muscle that was demonstrated by the professor was the latissimus dorsi, which attaches to the spinous processes of vertebrae T7 to L5 and the floor of the intertubercular sulcus. None of the other options describes attachments sites for muscles attaching to the upper limb.

During a gross anatomy laboratory session, a professor demonstrates a large back muscle that inserts onto the floor of the intertubercular sulcus of the humerus. Which of the following structures is most likely the vertebral origin of the muscle that the professor is demonstrating? A. Spinous processes of T7 to L5 B. Spinous processes of C7 to T12 C. Transverse processes of C1 to C4 D. Spinous processes of T2 to T5 E. Spinous processes of C7 and T1

*The answer is B.* The dorsal scapular nerve innervates the levator scapulae and rhomboid muscles, whereas the accessory nerve innervates the trapezius and sternocleidomastoid muscles. The serratus posterior superior is innervated by ventral primary rami of the spinal nerves, whereas the splenius cervicis and erector spinae are innervated by dorsal primary rami of the spinal nerves.

During a domestic dispute, a 16-year-old boy receives a deep stab wound around the superior angle of the scapula near the medial border, which injures both the dorsal scapular and spinal accessory nerves. Such an injury could result in paralysis or weakness of which of the following muscles? (A) Trapezius and serratus posterior superior (B) Rhomboid major and trapezius (C) Rhomboid minor and latissimus dorsi (D) Splenius cervicis and sternocleidomastoid (E) Levator scapulae and erector spinae

*The answer is B.* Lateral flexion is the best answer because other movements of the lumbar portion of the verte-bral column are very limited due to the orientation of the articular facets.

During a routine physical examination, a 65-yearold man is tested for ease and flexibility of the movements of his lumbar region. Which of the following movements is most characteristic of the intervertebral joints in the lumbar region? A. Circumduction B. Lateral flexion C. Abduction D. Adduction E. Inversion

*The answer is D.* Because the meninges and spinal cord are included in the protrusion, the patient's condition is a classic presentation of spina bifida with myelomeningocele. If the protrusion contains only meninges but no CNS tissue, it is known as spina bifida with meningocele. Meningitis is an inflammation of the meninges caused by bacteria, viral, or numerous other irritants (e.g., blood). It does not cause deformation of the vertebrae or result in protrusion of spinal cord contents. Spina bifida occulta is a normally asymp-tomatic condition in which the vertebral laminae fail to fuse completely during embryologic development. A tuft of hair is commonly seen growing over the affected region (usually lumbar in position).

Examination of a 3-day-old male infant reveals protrusion of his spinal cord and meninges from a defect in the lower back. Which of the following describes this congenital anomaly? A. Avulsion of meninges B. Meningitis C. Spina bifida occulta D. Spina bifida with myelomeningocele E. Spina bifida with meningocele

*The answer is A.* In the cervical region, the spinal nerve exits in the intervertebral foramen above the correspondingly named vertebrae. Therefore, the C3 spinal nerve exits above the C3 vertebrae and lies directly below the C2 vertebrae.

The following statement is written in the radiology report of an MRI scan of the cervical spine: "A large osteophyte is emanating from the posterolateral area of the vertebral body of the vertebra immediately above the C3 nerve root and is severely compressing the C3 nerve root." The osteophyte is most likely emanating from which of the following vertebrae? A. C2 B. C3 C. C4 D. C5 E. C1

*The answer is B.* The highest points of the iliac crests are used as a landmark for locating the position of L4 to L5 for a lumbar puncture; they are identified and traced medially toward the vertebral column (Tuffier's line). The inferior angles of the scapulae lie at vertebral level T7; the lowest ribs lead one to T12; the sacral hiatus is located lower at the distal portion of the sacrum; the posterior inferior iliac spines lie below S2.

When a lumbar puncture is performed to sample cerebrospinal fluid, which of the following external landmarks is the most reliable to determine the position of the L4 vertebral spine? A. The inferior angles of the scapulae B. The highest points of the iliac crests C. The lowest pair of ribs bilaterally D. The sacral hiatus E. The posterior inferior iliac spines

*The answer is A.* The anterior longitudinal ligament is a strong fibrous band that covers and connects the anterolateral aspect of the vertebrae and intervertebral discs; it maintains stability and prevents hyperextension. It can be torn by cervical hyperextension. The ligamentum flavum helps maintain upright posture by connecting the laminae of two adjacent vertebrae. The posterior longitudinal ligament runs within the vertebral canal supporting the posterior aspect of the vertebrae and prevents hyperflexion. The anulus fibrosus is the outer fibrous part of an intervertebral disc. The interspinous ligament connects adjacent spinous processes.

While waiting in his car at a stop sign, a 28-year old man was rear-ended by a van, resulting in neck hyperextension. He was admitted to the emergency department and a whiplash injury was diagnosed. The next day his neck was stiff and painful. Which structure was most likely damaged to cause the pain? A. Anterior longitudinal ligament B. Posterior longitudinal ligament C. Ligamentum flavum D. Intervertebral disc E. Supraspinous ligament

*The answer is C.* Spina bifida is a developmental condition resulting from incomplete fusion of the vertebral arches within the lumbar region. Spina bifida occulta commonly presents asymptomatically with midline, lumbar, cutaneous stigmata such as a tuft of hair and a small dimple. More severe forms (spina bifida cystica) are categorized into three types: Spina bifida cystica with meningocele presents with protrusion of the meninges through the unfused vertebral arches; spina bifida with myelomeningocele is characterized by protrusion both of the meninges and central nervous system (CNS) tissues and is often associated with neurologic deficits; and rachischisis, also known as spina bifida cystica with myeloschisis, results from a failure of neural folds to fuse and is characterized by protrusion of the spinal cord or spinal nerves and meninges.

A 1-year-old girl is brought to the clinic for a routine checkup. The child appears normal except for a dimpling of the skin in the midline of the lumbar region with a tuft of hair growing over the dimple. What is this relatively common condition that results from incomplete embryologic development? A. Meningomyelocele B. Meningocele C. Spina bifida occulta D. Spina bifida cystica E. Rachischisis

*The answer is A.* Lumbar puncture is generally performed at the level of L4 or L5. The supraspinous ligament extends between spinous processes on the dorsal aspect of the vertebrae. The needle will bypass this structure. The denticulate ligaments are not correct because they terminate with the conus medullaris at the level of L2 and are located laterally. The anterior longitudinal ligament extends along the most anterior aspect of the vertebral bodies and can be reached only ventrally. The posterior longitudinal ligament is present at the correct vertebral level but will be punctured only if the procedure is performed incorrectly as in this case, where hematopoietic cells were aspirated from the vertebral body anterior to the ligament. The nuchal ligament extends cranially from the supra-spinous ligament in the lower cervical region to the skull.

A 12-year-old child was brought to the emergency department by his parents because he has been suffering from a very high fever and severe stiffness in his back. The initial diagnosis is meningitis. The attending physician orders a lumbar puncture to confirm the diagnosis. Upon microscopic examination of the cerebrospinal fluid, hematopoietic cells are seen. Which of the following ligaments was most likely penetrated by the needle? A. Supraspinous B. Denticulate C. Anterior longitudinal D. Posterior longitudinal E. Nuchal ligament

*The answer is A.* Scoliosis (G: crookedness) is abnormal lateral and rotational curvature of the spine that may present with uneven hips, shoulders, and rib cage, a head that is not centered over the pelvis, the entire body leaning to one side, back pain, and/or fatigue. The given Anterior-Posterior (AP) X-ray shows an S-shaped curvature of the spine or vertebral column in this patient. In this X-ray, no apparent vertebral anomaly is apparent, so the diagnosis is most likely adolescent idiopathic scoliosis, which has an onset of 10 to 18 years of age and has no known cause. Most cases of adolescent idiopathic scoliosis (with curvatures of less than 20 degrees) require no treatment; however, if the curvature goes above 25 degrees, a back brace can be implemented to slow the progression of scoliosis.

A 12-year-old girl is examined by a school nurse who notices the girl's right scapula is more prominent than the left, her head is not centered directly over the pelvis, and her right hip is raised and more prominent. When the girl is asked to bend forward at the waist, the nurse observes asymmetry of the trunk. Which of the following diagnoses is most likely? (A) Scoliosis (B) Lordosis (C) Kyphosis (D) Osteoporosis (E) Osteoarthritis

*The answer is E.* Ligaments serve to restrict movement. The anterior longitudinal ligament courses downward on the anterior surface of the vertebral bodies attaching to the intervertebral discs along its way. It is stretches from the base of the skull inferiorly to the anterior surface of the sacrum. The anterior longitudinal ligament is the most anteriorly positioned ligament of the vertebral column and limits its extension. The posterior longitudinal ligament travels on the posterior surface of the vertebral bodies attaching to the inter- vertebral discs along the way. This ligament serves to prevent excessive flexion of the vertebral column and extends from C2 to the sacrum. The supraspinous ligament attaches the tips of the spinous processes to each other from C7 to the sacrum. Superiorly the liga- ment broadens becoming more distinct and triangular and is termed the ligamentum nuchae. Ligamentum nuchae limits excessive flexion of the cervical spine and serves as an attachment for muscles. Ligamentum flava attach the internal surfaces of adjacent lamina to each other and prevent them from pulling apart during flexion.

A 14-year-old girl accidentally flipped her bicycle off a curb, fell, and landed on her face. Although she was wearing a helmet, she landed in such a way that her neck was forced into hyperextension. Which of the following ligaments of the cervical spine was stretched to the greatest degree during her injury? A. Posterior longitudinal ligament B. Ligamentum nuchae C. Ligamenta flava D. Supraspinous ligament E. Anterior longitudinal ligament

*The answer is E.* CSF is found within the subarachnoid space and is continuous with the ventricles of the brain (CSF flows from the ventricles to the subarachnoid space). The epidural space, positioned between the dura mater and periosteum, contains fat and the internal vertebral venous plexus (of Batson). The subdural space, between the arachnoid mater and dura mater, exists only as a potential space and does not contain cerebrospinal fluid. The anterior and posterior longitudinal ligaments traverse the length of the vertebral bodies.

A 15-year-old woman was suspected of having meningitis. To obtain a sample of cerebrospinal fluid by spinal tap in the lumbar region (lumbar puncture), the tip of the needle must be placed in which of the following locations? A. In the epidural space B. Between anterior and posterior longitudinal ligaments C. Superficial to the ligamentum flavum D. Between arachnoid mater and dura mater E. In the subarachnoid space

*The answer is E.* Scoliosis is defined as a lateral deviation of the spinal column to either side and is often associated with a "rib-hump" as seen on examination when bending forward to touch the toes. Hyperkyphosis is an increased primary curvature of the spinal column. This curvature is associated with thoracic and sacral regions and is most likely this patient's clinical condition. Hyperlordosis is the increased secondary curvature affecting the cervical and lumbar regions.

A 16-year-old girl is sent for a presports physical examination prior to the beginning of her school year. She has no medical complaints or any clinical past history. On physical examination, the physician notices one shoulder is higher than the other. The student is then asked to bend forward at the waist to touch her toes. This maneuver results in a posterior bulging of the ribs on the right side. Which one of the following is most likely diagnosis? A. Kyphosis B. Spondylosis C. Lordosis D. Spondylolisthesis E. Scoliosis

*The answer is A.* The atlantoaxial joint is a synovial joint responsible for rotation of the head, not flexion, abduction, extension, or adduction. The atlanto-occipital joint is primarily involved in flexion and extension of the head on the neck.

A 22-year-old male soccer player is forced to leave the game following a head-to-head collision with another player. He is admitted to the hospital, and radiologic examination reveals slight dislocation of the atlantoaxial joint. As a result, he experiences decreased range of motion at that joint. What movement of the head would most likely be severely affected? A. Rotation B. Flexion C. Abduction D. Extension E. Adduction

*The answer is D.* Spondylolysis refers to a defect in the pars interarticularis of the affected vertebra (or L5 in this patient). Its meaning is derived from the Greek words "spondylos" (G: vertebra) and "lysis" (G: loosening). The given right posterior oblique X-ray reveals a fracture in the isthmus (or neck) of the pars interarticularis or the bony column formed by the superior and inferior articular processes of L5. This fracture has the appearance of a "headless Scottie Dog" in this image. To visualize the Scottie Dog, look at the intact L4 vertebra, and remember that the ear is formed by the superior articular process, the head is formed by the transverse process, its eye is the radiopaque pedicle, its foreleg is the inferior articular process, its body is represented by the lamina and spinous process, and the hind leg of the dog is the opposite inferior articular process. Because there is a fracture within the isthmus (or neck) of the pars interarticularis of L5, spondylolysis of L5 is the proper diagnosis.

A 16-year-old male soccer player was brought to the ER because of the acute lower back pain he experienced after performing a high-velocity kick of the ball. The given right posterior oblique X-ray reveals pathology within the L5 vertebra that has the appearance of a "headless Scottie Dog" when outlined. Based upon the X-ray results, which of the following diagnoses is the most likely cause of his lower back pain? (A) Ankylosing spondylitis (B) High-grade spondylolisthesis (C) Spondyloptosis (D) Spondylolysis (E) Spondylosis

*The answer is D.* The latissimus dorsi (L: widest muscle of back) is a large, fan-shaped muscle and a member of the superficial extrinsic layer of back muscles, which connect the upper limbs to the trunk. The chin-up movement described includes lifting the body toward the upper limb, which is a powerful extension action. In conjunction with the pectoralis major muscle, the latissimus dorsi muscle raises the trunk to the arm, which is crucial in performing chin-ups or climbing a tree. Remember that the superficial layer of back muscles is composed of upper limb muscles that take large bases of origin in the back. Athletes who make heavy use of extension and medial rotation of the arm (as in chin-ups or climbing in conditional exercises) typically have well-developed "lats" that give the classic "V" shape to the back.

A 17-year-old gymnast grips a high bar with his arms outstretched and begins to pull himself straight upward to the level of the bar, as in doing a chin-up. Which of the following muscles is the prime agonist in this action? (A) Serratus posterior superior (B) Rhomboid major (C) Levator scapulae (D) Latissimus dorsi (E) Longissimus

*The answer is C.* The splenius capitis is supplied by the dorsal rami. The obliquus capitis muscles are innervated by branches of the suboccipital nerve which also sup- plies the rectus capitus posterior major and minor. The greater occipital nerve supplies the semispinalis capitis.

A 22-year-old man has suffered from headaches and some muscle weakness to his upper muscles of the back for the last 6 months. An MRI shows a large tumor compressing the suboccipital and greater occipital nerves. Which of the following muscles will most likely still be functioning normally? A. Rectus capitis posterior major and minor B. Semispinalis capitis C. Splenius capitis D. Obliquus capitis superior E. Obliquus capitis inferior and lateral

*The answer is B.* The posterior longitudinal ligament is the only ligament spanning the posterior aspect of the vertebral bodies and intervertebral discs. With intervertebral disc herniation, the nucleus pulposus of the intervertebral disc protrudes posterolaterally. The anterior longitudinal ligament traverses the anterior side of the vertebral bodies and thus would not protect the spinal cord from direct compression. The supra- spinous and ligamentum flavum ligaments connect the spinous processes and the laminae of adjacent vertebrae, respectively. The nuchal ligament is a con-tinuation of the supraspinous ligaments near the C7 vertebrae and runs to the occipital protuberance.

A 19-year-old man is diagnosed with a herniated disc but he has no symptoms of spinal cord injury. In the event of intervertebral disc herniation in the cervical region, which of the following ligaments is in an anatomic position to protect the spinal cord from direct compression? A. Supraspinous B. Posterior longitudinal C. Anterior longitudinal D. Ligamentum flavum E. Nuchal ligament

*The answer is C.* The subarachnoid space, containing the CSF, is located between the pia and the arachnoid mater. Neither the epidural space, the subdural space, nor the pretracheal space contains CSF. Although the central canal, contained within the substance of the spinal cord, does contain CSF, extraction of CSF from this space would result in spinal cord injury. CSF circulates within the subarachnoid space and can be aspirated only from that location. The subdural space is only a potential space between the dura and arachnoid mater. The epidural space contains the epidural fat and Batson's venous plexus and is the site to inject an anesthetic for epidural anesthesia. CSF is not located in the pretracheal space.

A 19-year-old presents at the emergency department with high fever, severe headache, nausea, and stiff neck that have persisted for 3 days. The attending physician suspects meningitis and obtains a sample of CSF using a lumbar puncture. From which of the following spaces was the CSF collected? A. Epidural space B. Subdural space C. Subarachnoid space D. Pretracheal space E. Central canal of the spinal cord

*The answer is A.* During development the spinal cord fills the vertebral canal entirely. Due to differential growth of the vertebral column and the spinal cord, the cord ends at L3 in an infant. It gradually changes its position to the level of L1/L2, which is the adult level. S2 is the level at which the dural sac normally terminates.

A 2-month-old infant is admitted to the emergency department with symptoms of meningitis. A lumbar puncture is performed in order to examine the CSF to confirm the diagnosis. The needle is inserted into the lumbar cistern (dural sac). At which vertebral level will the conus medullaris typically be found in this patient? A. L3 B. L4 C. L5 D. S1 E. S2

*The answer is E.* The teres major is responsible for adduction and medial rotation of the humerus, the teres minor is responsible for lateral rotation of the humerus, the triceps brachii is responsible for extension of the forearm, the supraspinatus is responsible for the first 0 to 15 degrees of abduction, and the infraspinatus is a lateral rotator.

A 20-year-old male hiker suffers a deep puncture wound during a fall. Physical examination reveals a lesion between the trapezius and latissimus dorsi muscles on the right lateral side of his back. Upon admission to the hospital, physical examination reveals weak adduction and medial rotation of his arm. Which of the following muscles is most probably injured? A. Teres minor B. Triceps brachii C. Supraspinatus D. Infraspinatus E. Teres major

*The answer is A.* The suprascapular nerve passes through the suprascapular notch, deep to the superior transverse scapular ligament. This nerve is most likely affected in a fracture of the scapula as described in the question. The thoracodorsal nerve runs behind the axillary artery and lies superficial to the subscapularis muscle and would therefore be protected. The axillary nerve passes posteriorly through the quadrangular space, which is distal to the suprascapular notch. The subscapular nerve originates from the posterior cord of the brachial plexus, which is distal to the site of fracture.

A 22-year-old man is brought into the emergency department following a brawl in a tavern. He has severe pain radiating across his back and down his left upper limb. He supports his left upper limb with his right hand, holding it close to his body. Any attempt to move the left upper limb greatly increases the pain. A radiograph is ordered and reveals an unusual sagittal fracture through the spine of the left scapula. The fracture extends superiorly toward the suprascapular notch. Which nerve is most likely affected? A. Suprascapular nerve B. Thoracodorsal nerve C. Axillary nerve D. Subscapular nerve E. Suprascapular nerve and thoracodorsal nerve

*The answer is B.* The infraspinatus is responsible for lateral rotation of the humerus (along with the teres minor, not a choice here). The teres major is responsible for adduction and medial rotation of the humerus. The latissimus dorsi is responsible for adduction, exten-sion, and medial rotation of the humerus. The trapezius is an elevator of the scapula and rotates the scapula during abduction of the humerus above the horizontal plane. The supraspinatus is responsible for the first 0 to 15 degrees of abduction.

A 22-year-old man is thrown through a plate glass wall in a fight. Radiologic examination reveals that the lateral border of his right scapula is shattered. He is admitted to the emergency department, and physical examination reveals difficulty laterally rotating his arm. Which of the following muscles is most probably injured? A. Teres major B. Infraspinatus C. Latissimus dorsi D. Trapezius E. Supraspinatus

*The answer is D.* The space between the vertebral canal and the dura mater is the epidural space, which contains the internal vertebral venous plexus. The spinal cord and blood vessels lie deep to the pia mater. The space between the arachnoid and dura maters is the subdural space, which contains a film of fluid. The subarachnoid space contains cerebrospinal fluid (CSF).

A 23-year-old jockey falls from her horse and complains of headache, backache, and weakness. Radiologic examination would reveal blood in which of the following spaces if the internal vertebral venous plexus was ruptured? (A) Space deep to the pia mater (B) Space between the arachnoid and dura maters (C) Subdural space (D) Epidural space (E) Subarachnoid space

*The answer is B.* Caudal (epidural) anesthesia is used to block the spinal nerves in the epidural space by injecting local anesthetic agents via the sacral hiatus located between the sacral cornua. An intervertebral foramen transmits the dorsal and ventral primary rami of the spinal nerves. The vertebral canal accommodates the spinal cord. Dorsal and ventral sacral foramina transmit the dorsal and ventral primary rami of the sacral nerves.

A 24-year-old woman comes to a hospital to deliver her baby. Her obstetrician uses a caudal anesthesia during labor and childbirth to block the spinal nerves in the epidural space. Local anesthetic agents are most likely injected via which of the following openings? (A) Intervertebral foramen (B) Sacral hiatus (C) Vertebral canal (D) Dorsal sacral foramen (E) Ventral sacral foramen

*The answer is A.* When a lumbar puncture is performed, the needle must penetrate the ligamentum flavum, the dura mater, and finally the arachnoid mater to reach the subarachnoid space where the CSF is located. The lumbar cistern is a continuation of the subarachnoid space below the conus medullaris. The pia mater is adherent to the spinal cord, and the posterior longitudinal ligament is attached to the posterior aspect of the vertebral bodies.

A 24-year-old woman presents with severe headache, photophobia, and stiffness of her back. Physical examination reveals positive signs for meningitis. The attending physician decides to perform a lumbar puncture to determine if a pathogen is in the cerebrospinal fluid (CSF). What is the last structure the needle will penetrate before reaching the lumbar cistern? A. Arachnoid mater B. Dura mater C. Pia mater D. Ligamentum flavum E. Posterior longitudinal ligament

*The answer is D.* The rhomboid major and minor are supplied by the dorsal scapular nerve which also supplies the levator scapulae. The function of levator scapulae is elevation and inferior rotation of the scapulae. Abduc- tion of the arm above 90 degrees and protraction of the scapula are possible due to the action of serratus anterior, which is supplied by the long thoracic nerve. Medial rotation and adduction of the arm is performed mainly by the pectoralis major and latissimus dorsi, which also extends the arm. These are supplied by the medial and lateral pectoral nerves and thoracodorsal, respectively. Abduction of the arm through 0 to 15 degrees is produced by supraspinatus, which is supplied by the suprascapular nerve.

A 25-year-old male bodybuilder complains of difficulty moving his right shoulder for the past 2 weeks. Upon physical examination, the muscles of the left upper back and shoulder were notably larger than the right side. There was a notable decrease of muscle power on his right sided upper back and shoulder muscles when he was asked to pull the shoulder blades toward the middle of his back against resistance. Nerve conduction examination confirmed neurapraxia of the nerves supplying the rhomboid major and minor muscles. In which of the following functions will the bodybuilder most likely also demonstrate weakness? A. Abduction of the right arm above the horizontal level and protraction of the scapula B. Medial rotation and adduction of the right arm C. Extensions, adduction, and medial rotation of the right arm D. Elevation of the scapula and inferior rotation of the right shoulder E. Abduction of the right arm from 0 to 15 degrees

*The answer is B.* The anterior spinal artery is located anteriorly along the spinal cord and is not directly associated with the vertebrae. The vertebral arteries run through the transverse foramina of cervical vertebrae C6 through C1 and are therefore most closely associated with injury to the transverse processes. The ascending cervical artery is usually a very small branch from the thyrocervical trunk of the subclavian artery, running on the anterior aspect of the vertebrae. The deep cervical artery arises from the costocervical trunk and is also a very small artery and courses along the posterior aspect of the cervical vertebrae. The posterior spinal arteries are adherent to the posterior aspect of the spinal cord.

A 25-year-old male racing car driver is admitted to the emergency department after a severe car crash. Radiologic studies reveal damage to the tip of the transverse process of the third cervical vertebra, with a significantly large pulsating hematoma. What artery is the most likely to have been damaged? A. Anterior spinal artery B. Vertebral artery C. Ascending cervical artery D. Deep cervical artery E. Posterior spinal arteries

*The answer is B.* Spinal nerves exit the vertebral canal through the intervertebral foramina on the lateral sides of the vertebral column. Each intervertebral foramen is formed by the juxtaposing of the pedicles of successive articulated vertebrae. Thus, if the pedicles are compressed together, the intervertebral foramen is narrowed and a spinal nerve may be compressed ("pinched") as it traverses the intervertebral foramen. In the thoracic, lumbar, and sacral regions, each numbered segmental nerve passes through the intervertebral foramen below the matched numbered vertebra. Thus, the L3 spinal nerve traverses the intervertebral foramen below the L3 vertebra, between the L3 and L4 vertebrae. Because there is one more cervical spinal nerve (N = 8) than cervical vertebrae (N = 7), the cervical nerves exit the intervertebral foramina above the matching numbered vertebrae. Thus, the C3 spinal nerve passes through the opening between vertebrae C2 and C3. In this patient, the damage to the L3 spinal nerve on the left side is due to narrowing the intervertebral foramen caused by the compression of the pedicles of L2 and L3.

A 25-year-old medical student feels a sharp pain in his lower back while helping his landlady move a piano. After 3 days, the pain does not subside, so he receives a comprehensive examination from his physician that reveals a compression of the left side L3 spinal nerve. Which of the following is the most likely cause of this condition? (A) The pedicles of L2 and L3 are compressed together (B) The pedicles of L3 and L4 are compressed together (C) The laminae of L2 and L3 are compressed together (D) The laminae of L3 and L4 are compressed together (E) The spinous processes of L3 and L4 are compressed together

*The answer is D.* The cauda equina is the collection of dorsal and ventral roots of the lower lumbar and sacral spinal nerves below the spinal cord. Dorsal and ventral primary rami and dorsal roots of the thoracic spinal nerves and lumbar spinal nerves do not participate in the formation of the cauda equina.

A 25-year-old soldier suffers a gunshot wound on the lower part of his back and is unable to move his legs. A neurologic examination and magnetic resonance imaging (MRI) scan reveal injury of the cauda equina. Which of the following is most likely damaged? (A) Dorsal primary rami (B) Ventral primary rami (C) Dorsal roots of the thoracic spinal nerves (D) Ventral roots of the sacral spinal nerves (E) Lumbar spinal nerves

*The answer is E.* The atlantoaxial joints are synovial joints that consist of two plane joints and one pivot joint and are involved primarily in rotation of the head. Other movements do not occur at this joint.

A 26-year-old heavyweight boxer is punched on his mandible, resulting in a slight subluxation (dislocation) of the atlantoaxial joint. The consequence of the injury was decreased range of motion at that joint. What movement would be most affected? (A) Extension (B) Flexion (C) Abduction (D) Adduction (E) Rotation

*The answer is A.* The long thoracic is the only nerve that innervates the serratus anterior. The axillary nerve innervates the deltoid, the spinal accessory nerve innervates the sternocleidomastoid and trapezius, the dorsal scapular nerve supplies the rhomboid muscles and levator scapulae, and the latissimus dorsi is the muscle supplied by the thoracodorsal nerve.

A 35-year-old man is admitted to the emergency department after a severe car crash. After examining the patient the emergency medicine physician concludes that the serratus anterior muscle is damaged. Which of the following nerves innervates the serratus anterior muscle? A. Long thoracic B. Axillary C. Spinal accessory D. Dorsal scapular E. Thoracodorsal

*The answer is E.* The acromion (the highest point of the shoul-der) is the part of the scapula that forms the "point" of the shoulder. The coracoid process is located more medially. The superior angle of the scapula is located near the midline of the back. The glenoid of the scapula articulates with the head of the humerus to form the glenohumeral joint. The spine of the scapula is located posteriorly and separates supraspinous and infraspinous fossae.

A 26-year-old man painting his house slipped and fell from the ladder, landing on the pavement below. After initial examination in the emergency department, the patient is sent to the radiology department. Radiographs reveal that the portion of his left scapula that forms the tip, or point, of the shoulder has been fractured. Which part of the bone was fractured? A. Coracoid process B. Superior angle of the scapula C. Glenoid D. Spine of the scapula E. Acromion

*The answer is A.* The thoracodorsal nerve innervates the latissimus dorsi, one of major muscles that adduct and medially rotate the humerus. The axillary nerve supplies the deltoid muscle, the dorsal scapular nerve supplies the rhomboids and levator scapulae muscles, and the spinal accessory nerve innervates the trapezius. None of these nerves medially rotates or adducts the humerus. The radial nerve is responsible for the innervation on the posterior aspect of the arm and forearm. The medial and lateral pectoral nerves and the lower subscapular nerve supply the other medial rotators of the humerus.

A 27-year-old man is admitted to the emergency department after a car crash. Physical examination reveals weakness during medial rotation and adduction of the humerus. Which of the following nerves was most probably injured? A. Thoracodorsal B. Axillary C. Dorsal scapular D. Spinal accessory E. Radial

*The answer is D.* The body of vertebra T4 articulates with the heads of the fourth and fifth ribs. The body of the T3 vertebra articulates with the head of the third and fourth ribs. The neck of a rib does not articulate with any parts of the vertebra. The transverse process of the vertebra articulates with the tubercle of the corresponding rib. Therefore, the transverse process of vertebra T4 articulates with the tubercle of the fourth rib.

A 27-year-old mountain climber falls from a steep rock wall and is brought to the emergency department. His physical examination and computed tomography (CT) scan reveal dislocation fracture of the upper thoracic vertebrae. The fractured body of the T4 vertebra articulates with which of the following parts of the ribs? (A) Head of the third rib (B) Neck of the fourth rib (C) Tubercle of the fourth rib (D) Head of the fifth rib (E) Tubercle of the fifth rib

*The answer is C.* The vertebral canal is the longitudinal canal that extends through the vertebrae, containing the meninges, spinal cord, and associated ligaments. The internal vertebral venous plexus is the mostly valve- less network of veins extending longitudinally along the vertebral canal. Neither of these answer choices describes a specific space. The spinal epidural space is found superficially to the dura mater. It is a fat-filled space extending from C1 to the sacrum. The subarachnoid space is a true space containing CSF. It is found within the CNS and extends to the level of S2. The subdural space is a potential space between the dura and the arachnoid mater. Normally, these two layers are fused due to the pressure of CSF in the subarach-noid space.

A 28-year-old pregnant woman is admitted to the obstetrics department for delivery. In the final stages of labor, a caudal anesthetic is administered via the sacral hiatus. Into which of the following spaces in the sacral canal is the anesthetic placed? A. Vertebral canal B. Vertebral venous plexus C. Epidural space D. Subarachnoid space E. Subdural space

*The answer is B.* The anterior longitudinal ligament is a strong fibrous band that covers and connects the anterolateral aspect of the vertebrae and intervertebral discs; it maintains stability and prevents hyperextension. It can be torn by cervical hyperextension. The ligamentum flavum helps maintain upright posture by connecting the laminae of two adjacent vertebrae. The posterior longitudinal ligament runs within the vertebral canal supporting the posterior aspect of the vertebrae and prevents hyperflexion. The anulus fibrosus is the outer fibrous part of an intervertebral disc. The interspinous ligament connects adjacent spinous processes.

A 34-year-old woman is admitted to the emergency department after a car crash. Radiologic examination reveals a whiplash injury in addition to hyperextension of her cervical spine. Which of the following ligaments will most likely be injured? A. Ligamentum flavum B. Anterior longitudinal ligament C. Posterior longitudinal ligament D. Anulus fibrosus E. Interspinous ligament

*The answer is C.* The functions of the serratus anterior are protraction and rotation of the scapula. The rhomboid major and minor adduct the scapula, the serratus posterior inferior depresses the lower ribs, the levator scapulae elevates the scapula, and the latissimus dorsi adducts, extends, and medially rotates the arm.

A 35-year-old man is admitted to the emergency department after a severe car crash. After examining the patient the emergency medicine physician concludes that the serratus anterior muscle is damaged. Which of the following functions does the serratus anterior muscle serve? A. Adducts scapula B. Depresses ribs C. Protraction and rotation of scapula D. Elevation of scapula E. Adducts, extends, and medially rotates arm

*The answer is B.* The serratus anterior muscle pulls the scapula forward (protraction) and keeps the costal surface of the scapula closely opposed to the thoracic wall, preventing "winging" of the scapula. The levator scapulae elevates the scapula. The trapezius muscle is a powerful elevator of the shoulder and also rotates the scapula during reaching overhead. The rhomboid major and minor elevate and retract the scapula.

A 35-year-old man is admitted to the emergency department after a severe car crash. During physical examination of the patient the emergency medicine physician observes a winged scapula. Which of the following muscles is most likely injured? A. Levator scapulae B. Serratus anterior C. Trapezes D. Rhomboid major and minor E. Serratus posterior superior

*The answer is A.* The long thoracic nerve innervates the serratus anterior muscle, which protracts the scapula, holds the scapula close to the thoracic wall preventing it from "winging", and abducts and upwardly rotates the scapula. Patients with injury to this nerve will have their scapulae protruding on their back like a wing. The axillary nerve supplies the deltoid and teres minor muscles. The deltoid abducts, flexes, and extends and the teres minor laterally rotates the arm. The spinal accessory nerve is responsible for supplying the trapezius and sternocleidomastoid muscles. The trapezius elevates and upwardly rotates the scapula while the sternocleidomastoid flexes and pulls the chin upward to the opposite side. The dorsal scapular nerve supplies the rhomboid major and minor muscles and are responsible for retraction of the scapula. The thoracodorsal nerve supplies the latissimus dorsi muscle, which adducts, medially rotates, and extends the arm.

A 35-year-old man is admitted to the emergency department after a severe car crash. The emergency medicine physician examines the patient and observes what is shown in Figure 1-1. Which of the following nerves is most likely injured? A. Long thoracic B. Axillary C. Spinal accessory D. Dorsal E. Thoracodorsal

*The answer is E.* The T4 thoracic vertebra articulates with the head of the fifth rib. The head of the rib has two facets. The rib articulates with the superior facet on the body of its own vertebra (the fourth rib articulates with the superior facet T4 vertebra) and with the inferior facet on the body of the vertebra above (the fourth rib articulates with the inferior facet of T3 vertebra). Taking the T4 vertebra into consideration, the superior facet of this vertebra articulates with the head of the fourth rib and the inferior facet articulates with the head of the fifth rib. The head of the fourth rib has two points of articulation (a joint with the vertebral body and a costotransverse joint) on T4, so when it is injured it moves as a unit, whereas the fifth rib has only one articulation with T4.

A 35-year-old man pedestrian is crossing a busy intersection and is hit by a truck. He is admitted to the emergency department, and a CT scan reveals a dislocation of the fourth thoracic vertebra. Which of the following costal structures is most likely also involved in the injury? A. Head of the fourth rib B. Neck of the fourth rib C. Head of the third rib D. Tubercle of the third rib E. Head of the fifth rib

*The answer is A.* A rhizotomy is a neurosurgical procedure that selectively severs problematic spinal nerve roots to relieve pain or spastic paralysis (e.g., as often seen in cerebral palsy patients). The posterior roots are the only site where afferent (sensory) fibers are segregated from efferent (motor) fibers. In selective dorsal rhizotomy (SDR), severing the left C6-8 posterior roots could relieve the pain symptoms in the left forearm of this patient because they carry only sensory information.

A 35-year-old woman suffers intractable pain in her left forearm. Neurosurgical consultation leads to a decision to conduct a rhizotomy to relieve the condition. At which of the following locations is the rhizotomy best performed to relieve the patient's pain? (A) Posterior (dorsal) roots (B) Posterior (dorsal) primary rami (C) Spinal nerves (D) Anterior (ventral) roots (E) Anterior (ventral) primary rami

*The answer is D.* The boundaries of an intervertebral foramen (clockwise) include the following: the superior margin (roof) is formed by the inferior vertebral notch of the vertebra above, the anterior margin by the interverte-bral disc between the vertebral bodies of the adjacent vertebrae, the inferior margin (floor) by the superior vertebral notch of the vertebra below, and the posterior margin by the zygapophysial (facet) joint of the adjacent vertebrae. Each pedicle contains superior and inferior vertebral notches.

A 38-year-old man who is a professional golfer complains of chronic lower back pain with radiating pain to the heel. The pain is so debilitating that he now has trouble ambulating. MRI of the lower back reveals severe narrowing of an intervertebral foramen (IVF), which has caused compression of the exiting nerve root. Surgery is required to correct the problem. During surgery the neurosurgeon carefully accesses the IVF using a lateral approach and shaves bone off the superior margin (roof) of the IVF to decompress the exiting nerve root. Which of the following vertebral bony features is the neurosurgeon most likely shaving off? A. Superior articular process B. Lamina C. Inferior articular process D. Pedicle E. Spinous process

*The answer is E.* The ligamentum flavum lies within the vertebral canal on the anterior aspect of the vertebral arches connecting the lamina of adjacent vertebrae. Puncturing this ligament allows the needle to enter into the epidural/extradural space for the injection of the anesthetic. Although the posterior longitudinal ligament lies within the spinal canal, it will not be punctured during the procedure. The supraspinous ligament connects and passes along the tips of the vertebral spinous processes. The interspinous liga- ment lies between adjacent spinous processes. The anterior longitudinal ligament connects the anterior aspect of the vertebral body. These ligaments do not lie within the vertebral canal.

A 38-year-old woman has been in labor for 14 hours and has agreed to have an epidural anesthetic injection for pain control. Which of the following structures is most likely to be the last penetrated by the needle before it reaches the epidural space? A. Supraspinous ligament B. Interspinous ligament C. Anterior longitudinal ligament D. Posterior longitudinal ligament E. Ligamenta flava

*The answer is C.* The trapezius receives blood from the superficial branch of the transverse cervical artery. The latissimus dorsi receives blood from the thoracodorsal artery. The rhomboid major receives blood from the deep or descending branch of the transverse cervical artery. The multifidus and longissimus capitis receive blood from the segmental arteries.

A 38-year-old woman with a long history of shoulder pain is admitted to a hospital for surgery. Which of the following muscles becomes ischemic soon after ligation of the superficial or ascending branch of the transverse cervical artery? (A) Latissimus dorsi (B) Multifidus (C) Trapezius (D) Rhomboid major (E) Longissimus capitis

*The answer is A.* The anterior longitudinal ligament lies anterior to the vertebral bodies along the vertebral column. The ligamentum flavum connects the laminae of two adjacent vertebrae. The nuchal ligament is a continuation of the supraspinous ligament above C7, which connects spinous processes. The posterior longitudinal ligament lies on the posterior margin of the vertebral bodies. The transverse cervical (cardinal) ligament is associated with the pelvic region of the body and not the spinal column

A 39-year-old man presents with severe neck pain after a whiplash injury, sustained when his car was struck from behind. Radiologic studies reveal trauma to the ligament lying on the anterior surface of the cervical vertebral bodies. Which ligament is most likely disrupted? A. Anterior longitudinal ligament B. Ligamentum flavum C. Nuchal ligament D. Posterior longitudinal ligament E. Transverse cervical ligament

*The answer is C.* The long thoracic nerve innervates the serratus anterior, which is responsible for elevation and protraction of the scapula beyond the horizontal level while maintaining its position against the thoracic wall. Along with the thoracodorsal nerve, the long thoracic nerve runs superficially along the thoracic wall and is subject to injury during a mastectomy procedure. The axillary nerve, the spinal accessory nerve, and the thoracodorsal nerve supply the deltoid muscle, trapezius muscle, and latissimus dorsi muscles, respectively. The dorsal scapular nerve is responsible for innervation of the rhomboids and levator scapulae. Aside from the long thoracic and thoracodorsal nerves, the remaining nerves do not course along the lateral thoracic wall.

A 39-year-old woman complains of an inability to reach the top of her head to brush her hair with her right hand. History reveals that she had undergone a mastectomy procedure of her right breast 2 months earlier. Physical examination demonstrates winging of her right scapula. Which nerves were most likely damaged during surgery? A. Axillary B. Spinal accessory C. Long thoracic D. Dorsal scapular E. Thoracodorsal

*The answer is C.* A posterolateral herniation of the intervertebral disk at disk level L4-L5 affects the fifth lumbar nerve not the fourth lumbar nerve. The first seven cervical nerves exit above the corresponding vertebra, and the eighth cervical nerve exits below the seventh cervical vertebra because there are eight cervical nerves but only seven cervical vertebrae. The rest of the spinal nerves exit below their corresponding vertebrae.

A 39-year-old woman with headaches presents to her primary care physician with a possible herniated disk. Her magnetic resonance imaging (MRI) scan reveals that the posterolateral protrusion of the intervertebral disk between L4 and L5 vertebrae would most likely affect nerve roots of which of the following spinal nerves? (A) Third lumbar nerve (B) Fourth lumbar nerve (C) Fifth lumbar nerve (D) First sacral nerve (E) Second sacral nerve

*The answer is A.* The odontoid process, or the dens, projects superiorly from the body of the axis and articulates with the anterior arch of the atlas. The posterior and anterior tubercles of the atlas are bony eminences on the outer surface. The inferior articular facet is where the axis joins to the C3 vertebra. A, Lateral radiograph shows that this patient has only mild prevertebral swelling, which is centered at the odontoid (see arrowheads in Fig. 1-5, p. 20). The odontoid is displaced posteriorly relative to the C2 body (arrow) and is angled posteriorly. These findings indicate a fracture. B, The fracture is extremely subtle on the open-mouth odontoid radiograph (arrows). C, Sagittal CT reconstruction shows the fracture.

A 40-year-old woman survived a car crash in which her neck was hyperextended when her vehicle was struck from behind. At the emergency department a plain radiograph of her cervical spine is shown below (Fig. 1-5). Which of the following was also most likely injured? A. Anterior arch of the atlas B. Posterior tubercle of the atlas C. Atlanto-occipital joint D. Inferior articular process of the axis E. Anterior tubercle of the atlas

*The answer is A.* The odontoid process, or the dens, projects superiorly from the body of the axis and articulates with the anterior arch of the atlas. The posterior and anterior tubercles of the atlas are bony eminences on the outer surface. The inferior articular facet is where the axis articulates with the C3 vertebra.

A 40-year-old woman survived a car crash in which her neck was hyperextended when her vehicle was struck from behind. At the emergency department, a plain radiograph of her cervical spine revealed a fracture of the odontoid process (dens). Which of the following was also most likely injured? A. Anterior arch of the atlas B. Posterior tubercle of the atlas C. Atlanto-occipital joint D. Inferior articular process of the axis E. Anterior tubercle of the atlas

*The answer B.* The thoracodorsal nerve innervates the latissimus dorsi, which has no direct action on the shoulder girdle. The spinal accessory nerve is the eleventh cranial nerve (CN XI) and innervates both the trapezius and sternocleidomastoid muscles. The loss of CN XI results in drooping of the shoulder due to paralysis of the trapezius. In addition to the clinical findings of the MRI, one can test the innervation of this nerve by asking the patient to shrug his or her shoulders against resistance (testing the trapezius), as well as turning his or her head against resistance (testing the sternocleidomastoid). The dorsal scapular nerve usually innervates the levator scapulae muscle and the rhomboid muscles. The greater occipital nerve is primarily a sensory nerve innervating the posterolateral aspect of the scalp. The axillary nerve is a branch of the brachial plexus and innervates the deltoid and teres minor muscles. It is not involved in shoulder elevation.

A 42-year-old woman complains of pain and stiffness in her neck. She was injured sliding into second base headfirst during her company's softball game. Radiographs reveal no fractures of her spine. However, upon physical examination, her right shoulder is drooping and she has difficulty in elevating that shoulder. If you ordered an MRI, it would most likely reveal soft tissue damage involving which of the following nerves? A. Thoracodorsal nerve B. Spinal accessory nerve C. Dorsal scapular nerve D. Greater occipital nerve E. Axillary nerve

*The answer is C.* The anterior longitudinal ligament runs along the anterior-most aspect of the vertebral column from C1 to the sacrum and would therefore be unaffected by a laminectomy. Denticulate ligaments extend laterally from the pia mater to the arachnoid mater along the length of the spinal cord. The ligamentum flavum is one of the two ligaments found in the vertebral canal and is adherent to the anterior aspect of the vertebral arches and often greatly thickened in spinal pathology. It is thus simultaneously removed upon excision of the lamina. The nuchal ligament is a thick longitudinal extension continuing from the supraspinous ligament at the level of C7 to the external occipital protuberance (inion). The cruciate ligament is an incorrect answer because it is located anterior to the spinal cord, and thus would not be involved in laminectomy.

A 42-year-old woman is diagnosed with stenosis of the cervical vertebral canal. A laminectomy of two vertebrae is performed. Which of the following ligaments will most likely also be removed? A. Anterior longitudinal B. Denticulate C. Ligamentum flavum D. Nuchal E. Cruciate

*The answer D.* All cervical spinal nerves exit through the intervertebral foramina above the corresponding vertebrae, except the eighth cervical nerves, which run inferior to the seventh cervical vertebra. All other spinal nerves exit the intervertebral foramina below the corresponding vertebrae. Therefore, the fifth cervical nerve passes between the fourth and fifth cervical vertebrae and the fourth thoracic nerve runs between the fourth and fifth thoracic vertebrae.

A 42-year-old woman with metastatic breast cancer is known to have tumors in the intervertebral foramina between the fourth and fifth cervical vertebrae and between the fourth and fifth thoracic vertebrae. Which of the following spinal nerves may be damaged? (A) Fourth cervical and fourth thoracic nerves (B) Fifth cervical and fifth thoracic nerves (C) Fourth cervical and fifth thoracic nerves (D) Fifth cervical and fourth thoracic nerves (E) Third cervical and fourth thoracic nerves

*The answer is D.* All of the spinal nerves from C6 and below will be affected. The trapezius and sternocleidomastoid muscles will be intact because they are innervated by the spinal accessory nerve. The deltoid will be affected because its nerve motor supply is from the axillary nerve derived from C5 and C6. The diaphragm will work properly as its motor nerve supply is derived from the phrenic nerve (C3 to C5).

A 43-year-old male construction worker survived a fall from a two-story building but lost all sensation in his lower limbs and was admitted to the hospital for examination and treatment. Radiologic studies revealed that he crushed his spinal cord at vertebral level C6. Which of the following muscles will most likely be paralyzed? A. Sternocleidomastoid B. Trapezius C. Diaphragm D. Latissimus dorsi E. Deltoid

*The answer is C.* Spina bifida occulta is the mildest form of spina bifida (L: split spine). In this developmental disorder, the left and right neural arch elements fail to fuse completely in the dorsal midline. However, the split in the vertebra is so small that the meninges and elements of the spinal cord do not protrude through the defect. In the given plain film, the black arrow indicates a failure of fusion of the laminae of L5, producing a cleft. Individuals with spina bifida occulta may possess a tuft of hair and/or dimple in the skin overlying the affected vertebral levels, as noted in this patient. Other individuals with this condition may have no visible evidence, a lipoma, or even a birthmark in the overlying skin of the affected region. Some studies suggest approximately 10% of the general population have this mildest form of spina bifida. In this case, the X-ray verifies the diagnosis of spina bifida occulta. Because this condition is normally clinically asymptomatic and unnoticed, it is seemingly "hidden" (occult). In fact, most research suggests no relationship between spina bifida occulta and back pain.

A 45-year-old man goes to his family physician complaining of lower back pain after spending a weekend clearing trees off his property. During examination, the doctor notes a tuft of hair and a dimple on the skin of the patient's lower back. A plain film reveals a congenital defect in his L5 vertebra, indicated by the black arrow in the given X-ray. No other structural abnormalities are noted. Based on these findings, what is the most likely diagnosis for this patient? (A) Meningocele (B) Anencephaly (C) Spina bifida occulta (D) Spina bifida cystica (E) Spina bifida with myeloschisis

*The answer is C.* The weakness in shoulder movement results from denervation of the teres minor and deltoid by the axillary nerve, which passes through the quadrangular space. Quadrangular space syndrome occurs when there is hypertrophy of the muscles that border the quadrangular space or fibrosis of portions of the muscles that are in contact with the nerve.

A 45-year-old woman is admitted to the outpatient clinic for shoulder pain. During physical examination she presents with weakened shoulder movements. Radiologic examination reveals signs of quadrangular space syndrome, causing weakened shoulder movements. Which of the following nerves is most likely affected? A. Suprascapular B. Subscapular C. Axillary D. Radial E. Ulnar

*The answer is E.* Disc herniation in the lumbar region between L4 and L5 affects the L5 spinal nerve roots. Even though the L4 spinal nerve root lies directly between the L4 and L5 vertebrae, it exits from the spinal canal superior to the intervertebral disc, whereas the L5 spinal nerve root lies directly posterior to the disc.

A 45-year-old woman states that she has experienced moderate pain for 2 years over her left lower back; pain that radiates to her left lower limb. She states that after lifting a case of soft drinks, the pain suddenly became intense. She was admitted to the emergency department. Radiologic examination revealed intervertebral disc herniation between vertebral levels L4 and L5. Which of the following nerves was most likely affected by the disc herniation? A. L1 B. L2 C. L3 D. L4 E. L5

*The answer is C.* The C1 vertebra, or atlas, is normally a closed ring with no vertebral body. Excessive vertical, or downward, force on the top of the head can fracture the anterior and posterior arches of C1 in multiple places, leading to a Jefferson (burst) fracture of C1. Due to the vertical force of the concrete mix striking the top of the man's head, the lateral masses of C1 are driven laterally due to the oblique articulation between the occipital condyles and the superior articular processes of the lateral masses of C1. This vertical compression force fractures the anterior and posterior arches of C1 bilaterally, as confirmed by the axial CT scan. Jefferson fractures of C1 often occur with axial loading force when the top of the head is impacted by a hard or heavy object. Because the fractures within the bony ring of the atlas actually increase its dimension, this type of fracture does not usually result in spinal cord injury; however, upper neck pain would be present. The axial CT scan confirms the diagnosis of a Jefferson (burst) fracture of C1.

A 46-year-old supervisor was reading a work order on a construction site when a 60 lb bag of concrete mix was accidentally dropped on the apex of his head. He was immobilized and brought to the ER where he presented with upper neck pain but no neurological signs. Based upon the given axial CT scan and the patient's presentation, which of the following diagnoses is most likely? (A) Damage to the cervical spinal cord (B) No pathology is apparent on the CT scan (C) Jefferson (burst) fracture of C1 (D) Fracture of the dens axis (odontoid process) (E) Atlanto-axial subluxation

*The answer is B.* Spondylolisthesis is an anterior vertebral displacement created by an irregularity in the anterior margin of the vertebral column such that L5 and the overlying L4 (and sometimes L3) protrude forward rather than being restrained by S1. Spondylolysis is a condition in which the region between the superior and inferior articular facets (on the posterior arch of the L5 vertebra) is damaged or missing, which is not the case in this example. Herniation is a protrusion of the nucleus pulposus through the anulus fibrosus, and this is not associated with vertebral dislocation. Hyperlordosis and scoliosis are excessive curvatures that do not involve dislocations.

A 53-year-old man is admitted to the emergency department due to severe back pain. MRI examination reveals anterior dislocation of the body of the L5 vertebra upon the sacrum. Which of the following is the most likely diagnosis? A. Spondylolysis B. Spondylolisthesis C. Herniation of intervertebral disc D. Lordosis E. Scoliosis

*The answer is A.* A fracture of the pars interarticularis is termed spondylolysis. Spondylolisthesis is when the anterior portion of the vertebra is displaced after fracture of the pars interarticularis. A herniated disc is when the nucleus pulposus protrudes through the anulus fibrosus. Lordosis is the normal curvature of the cervical and lumbar spine. Scoliosis is an abnormal lateral curvature of the spine which usually also has a degree of rotation of the vertebrae.

A 53-year-old man is admitted to the emergency department with severe back pain. MRI examination reveals fracture of the pars interarticularis and normal alignment of the body of the L5 vertebra upon the sacrum. What is the most likely diagnosis? A. Spondylolysis B. Spondylolisthesis C. Herniation of intervertebral disc D. Lordosis E. Scoliosis

*The answer is B.* The transverse ligament of the atlas anchors the dens laterally to prevent posterior displacement of the dens, which has been torn in this injury. The anterior longitudinal ligament runs on the anterior aspect of the vertebrae and is not affected. The ligamentum flavum is found on the posterior aspect of the vertebral canal and does not contact the anteriorly placed dens. The supraspinous ligament is located along the spinous processes of the vertebrae. The nuchal ligament is a longitudinal extension of the supraspinous and interspinous ligaments above the level of C7.

A 53-year-old man was in a head-on collision resulting in the dens crushing the spinal cord. Which ligament was most likely torn for the dens to crush the spinal cord? A. Anterior and posterior longitudinal ligaments B. Transverse ligament of the atlas C. Interspinous ligament D. Supraspinous ligament E. Nuchal ligament

*The answer is B.* The anterior longitudinal ligament runs on the anterior aspect of the vertebrae and is not affected. The transverse ligament of the atlas anchors the dens laterally to prevent posterior displacement of the dens. This ligament has been torn in this injury. The ligamentum flavum is found on the posterior aspect of the vertebral canal and does not contact the ante-riorly placed dens. The supraspinous ligament is located along the spinous processes of the vertebrae. The nuchal ligament is a longitudinal extension of the supraspinous ligament above the level of C7.

A 53-year-old man was in a head-on vehicle collision that resulted in compression of his spinal cord by the dens (odontoid process) of the axis, with resulting quadriplegia. Which of the following ligaments was most probably torn? A. Anterior longitudinal ligament B. Transverse ligament of the atlas C. Ligamentum flavum D. Supraspinous ligament E. Nuchal ligament

*The answer is B.* Hyperkyphosis is characterized by a "hunchback" due to an abnormal increase in curvature of the thoracic region of the vertebral column. Hyperlordosis, or "swayback," is an increase in lumbar curvature of the spine. Lordosis can be physiologic, such as seen in a pregnant woman. Scoliosis is a lateral curvature of the spine with rotation of the vertebrae. Spina bifida is a neural tube defect characterized by failure of closure of the vertebral arch. Osteoarthritis is a degenerative disorder that affects the articular cartilage

A 55-year-old man with severe coughing is admitted to the hospital. Radiologic examination is consistent with tuberculosis of the right lung, with extension to the thoracic vertebral bodies of T6 and T7, producing a "gibbus deformity." Which of the following conditions is most likely also to be confirmed by radiologic examination? A. Hyperlordosis B. Hyperkyphosis C. Scoliosis D. Spina bifida E. Osteoarthritis

*The answer is A.* The region bounded by the upper border of the latissimus dorsi, the lateral border of the trape-zius, and the medial border of the scapula is known as the triangle of auscultation. Lung sounds can be heard most clearly from this location because minimal tissue intervenes between the skin of the back and the lungs. The deltoid, levator scapulae, and trapezius do not form the borders of the so-called "triangle of auscultation." The latissimus dorsi, external abdominal oblique, and iliac crest form the border of Petit's inferior lumbar triangle. The quadratus lumborum, internal abdominal oblique, and inferior border of the twelfth rib form the border of the Grynfeltt's superiolumbar triangle. The rectus abdominis, inguinal ligament, and inferior epigastric vessels form the border of the inguinal triangle (of Hesselbach).

A 6-year-old boy is admitted to the hospital with coughing and dyspnea. During taking of the history, he complains that it feels like there is glass in his lungs. Auscultation reveals abnormal lung sounds. The abnormal lung sounds are heard most clearly during inhalation with the scapulae protracted. Which of the following form the borders of a triangular space where one should place the stethoscope in order to best hear the lung sounds? A. Latissimus dorsi, trapezius, medial border of scapula B. Deltoid, levator scapulae, trapezius C. Latissimus dorsi, external abdominal oblique, iliac crest D. Quadratus lumborum, internal abdominal oblique, inferior border of the twelfth rib E. Rectus abdominis, inguinal ligament, inferior epigastric vessels

*The answer is B.* Compression of nerves at the intervertebral foramen indicates a disc herniation. A disc herniation is characterized by protrusion of the nucleus pulposus through the anulus fibrosus posterolaterally into the spinal canal or intervertebral foramen. In general, the ligaments may be affected by the herniation but are not responsible for the compression of the spinal nerve roots.

A 65-year-old man complains of severe back pain and the inability to move his left lower limb. Radiologic studies demonstrate compression of nerve elements at the intervertebral foramen between vertebrae L5 and S1. Which structure is most likely responsible for this space-occupying lesion? A. Anulus fibrosus B. Nucleus pulposus C. Posterior longitudinal ligament D. Anterior longitudinal ligament E. Ligamentum flavum

*The answer is B.* A crush fracture is characterized by compression of the entire vertebral body. The wedge fracture is similar in that it affects the vertebral bodies, but it involves small fractures around the perimeter of the vertebral body. Both of these fractures cause reductions in overall height. Fracture of the spinal, transverse, or superior articular processes can be due to an oblique, transverse, or comminuted fracture. Intervertebral discs are associated with disc herniation, not compression fractures.

A 68-year-old man is admitted to the hospital due to severe back pain. Radiologic examination reveals severe osteoporosis of the vertebral column, with compression fractures to vertebrae L4 and L5. Which of the following parts of the vertebrae are most likely to be fractured in this patient? A. Spinous process B. Vertebral bodies C. Transverse process D. Superior articular process E. Intervertebral disc

*The answer is E.* Kyphosis (hunchback or humpback) is an abnormally increased thoracic curvature, usually resulting from osteoporosis. Lordosis is an abnormal accentuation of the lumbar curvature. Spina bifida occulta is failure of the vertebral arch to fuse (bony defect only). Meningocele is a protrusion of the meninges through the unfused arch of the vertebra, whereas meningomyelocele is a protrusion of the spinal cord and the meninges.

A 69-year-old man has an abnormally increased curvature of the thoracic vertebral column. Which of the following conditions is the most likely diagnosis? (A) Lordosis (B) Spina bifida occulta (C) Meningocele (D) Meningomyelocele (E) Kyphosis

*The answer is D.* The diaphragm is innervated by the phrenic nerve, which arises from C3 to C5. The rhomboid, serratus anterior, supraspinatus, and latissimus dorsi are innervated by the ventral rami of the brachial plexus (C5 to T1).

A 69-year-old woman visits her physician due to severe neck pain. Radiologic studies reveal bony growths (osteophytes) in the intervertebral foramen between vertebrae C2 and C3. Which of the following muscles would be most likely affected by this condition? A. Rhomboideus major B. Serratus anterior C. Supraspinatus D. Diaphragm E. Latissimus dorsi

*The answer is E.* In the cervical region, spinal nerves exit the vertebral column above their named vertebrae. From the thoracic region and below the spinal nerves exit the vertebral column below their named vertebrae.

A 70-year-old man with prostate cancer is experiencing sharp shooting pains radiating from his neck into the upper limb. An MRI of his spine demonstrates a small metastatic mass in the cervical region extending into the left intervertebral foramen between C6 and C7. The intervertebral discs appear normal. Which neural structure is most likely being compressed by the metastatic mass to account for the pain? A. C8 spinal nerve B. Dorsal horn of C6 spinal cord segment C. C6 spinal nerve D. Dorsal horn of C7 spinal nerve E. C7 spinal nerve

*The answer is D.* Cervical spinal stenosis describes a narrowing of the vertebral canal that is typically seen in older individuals with degenerative changes in the cervical spine. In the given MRI, the vertebral canal, which houses the spinal cord, is impinged by intervertebral disc degeneration in the midcervical region, which results in no cerebral spinal fluid (CSF), which appears as a hyperintense (white) signal in this MRI, being visible anterior or posterior to the spinal cord. CSF can be seen surrounding the spinal cord above and below the midcervical region. In this patient, the tingling numbness of the fourth and fifth digits within his right hand is exacerbated by hyperextension of the cervical vertebrae (or looking up), which implies the spinal cord is further compressed by hyperextension of the neck leading to myelopathy (damage to the spinal cord itself). Due to the age of the patient, the degenerative nature of the cervical vertebrae, and the absence of trauma, spinal cord stenosis is the most likely diagnosis, and this condition was confirmed with the given T2-weighted MRI.

A 73-year-old man comes to his physician complaining of a tingling numbness within the fourth and fifth digits of his right hand. These symptoms are exacerbated when he looks up at the ceiling. The given sagittal T2-weighted MRI reveals degenerative changes in his cervical vertebrae. Which of the following diagnoses is confirmed by the MRI and would result in the symptoms of this patient? (A) Burst fracture of C3 vertebral body (B) Ruptured anterior longitudinal ligament (C) Transection of the cervical spinal cord (D) Cervical spinal stenosis (E) Traumatic spondylolisthesis of C5

*The answer is B.* Scoliosis is defined as a lateral deviation of the spinal column to either side. Hyperkyphosis is an increased primary curvature of the spinal column. This curvature is associated with thoracic and sacral regions and is most likely this patient's clinical condi-tion. Spinal stenosis is a narrowing of the vertebral canal and is not directly associated with a displace-ment of the spinal column. Hyperlordosis is the increased secondary curvature affecting the cervical and lumbar regions. A herniated disc is a rupture of the anulus fibrosus of the intervertebral disc, com-monly causing a posterolateral displacement of the nucleus pulposus into the vertebral canal.

A 79-year-old man, a retired military veteran, presents to the outpatient clinic with an abnormal curvature of the vertebral column. He complains that it has become increasingly painful to walk around town. Upon physical examination, he has an abnormally increased convexity to his thoracic curvature resulting from osteoporosis. Which of the following is the most likely clinical condition of this patient's spine? A. Scoliosis B. Hyperkyphosis C. Spinal stenosis D. Lordosis E. Herniated disc

*The answer is C.* Spondylolysis means a defect in the pars interarticularis of the affected vertebra (or C2 in this patient). The lateral cervical X-ray of this patient reveals a bilateral fracture of the axis (or C2) in the pars interarticularis or the bony column formed by the superior and inferior articular processes of C2. This type of cervical fracture is often seen after hyperextension of the head in relationship to the neck, and it is called a traumatic spondylolysis of C2 or hangman fracture. It receives its alternative name, hangman fracture, because it is often seen in criminals executed by hanging due to the hyperextension of the head due to the placement of the noose under their chin. This lateral cervical film also reveals a traumatic spondylolisthesis of C2, which is sometimes evident following an acute fracture of the pars interarticularis of C2. Spondylolisthesis describes the anterior displacement of a vertebra in relation to the vertebra below it, and its meaning is derived from the Greek words "spondyl" (G: vertebrae) and "olisthesis" (G: slip). In the given X-ray, the vertebral body of C2 is displaced anterior in relationship to C3, which confi rms the traumatic spondylolisthesis of C2. Therefore, the X-ray clearly shows a bilateral fracture of the pars interarticularis of C2 (or traumatic spondylolysis of C2 [hangman fracture]) and spondylolisthesis, anterior displacement, of C2 in relationship to the vertebral body of C3. This specific fracture was classifi ed in 1981 by Effendi into a subtype called "forward dislocation, axial arch fracture, Effendi Type II."

A helmet-less 24-year-old man was riding an all-terrain vehicle (ATV or quad bike) when a thin, unseen, horizontal clothes line impacted the man's neck under his chin, knocking him backward off the ATV. Due to his forward momentum at the time of impact, his head was hyperextended on his neck. Emergency medical technicians (EMTs) established his airway, stabilized his neck, and brought him to the emergency room. A lateral cervical X-ray revealed pathology within the cervical vertebrae. Based upon the nature of the accident and the evidence of the given X-ray, which of the following diagnoses is most probable? (A) Ruptured anterior longitudinal ligament (B) Jefferson (burst) fracture of C1 (C) Traumatic spondylolysis of C2 (D) Fracture of the dens axis (odontoid process) (E) Atlanto-axial subluxation

*The answer is E.* Sensory receptors in the skin overlying the trapezius muscle project through general sensory (general somatic afferent) neurons to the spinal cord via the posterior (dorsal) primary rami of spinal nerves (marked "E" in this diagram), traverse the mixed spinal nerves, travel within posterior (dorsal) roots of spinal nerves, and reach the posterior (dorsal) gray horn of the spinal cord. Cutting the posterior rami of spinal nerves would cause degeneration of the distal axonal processes of the general sensory fibers and lead to loss of sensation in the skin of the back. Additionally, the distal axonal processes of somatic motor (general somatic efferent or GSE) neurons and visceral motor (general visceral efferent or GVE) neurons contained within the posterior primary rami would be damaged as well, causing motor and autonomic deficits in the back, respectively. Remember that the trapezius muscle is a component of the superficial extrinsic layer of back muscles, which connect the upper limbs to the trunk. These muscles are innervated by anterior primary rami of spinal nerves, except for the trapezius, which is supplied by the accessory nerve (CN XI). However, the skin overlying the trapezius muscle is innervated by the posterior (dorsal) primary rami of spinal nerves.

A lesion of which of the indicated structures would cause loss of sensation in the skin overlying the trapezius muscle?

*The answer is A.* The objective of a lumbar puncture is to enter (tap) the subarachnoid space and access the CSF. This procedure is performed using a long spinal needle. For several reasons, this procedure is best performed in the low lumbar region, between the spinous processes of the L3 and L4 (sometimes L4 and L5) vertebrae. The medullary cone (or conus medullaris) is the tapered terminal end of the spinal cord. In adults, the medullary cone is normally located within the T12-L3 vertebral levels. Thus, penetrating the vertebral canal and subarachnoid space below L3 is the "safe" place to go, in that the spinal needle should not penetrate the spinal cord.

A physician orders a lumbar puncture (spinal tap) for his 43-year-old female patient in order to obtain a sample of cerebrospinal fluid (CSF). He explains to her that this procedure will be done in the lower back, between the spinous processes of the L3 and L4 vertebrae. What is the best reason for performing the lumbar puncture at this location? (A) The medullary cone ends at or above the L3 level (B) The subarachnoid space ends at the L3 level (C) The intervertebral foramina at L3-4 are large and easy to penetrate (D) No vertebral venous plexuses exist below the L3 level (E) The ligamenta flava are absent below the L3 level

*The answer is A.* Approximately 90% of intervertebral disc herniations occur toward the bottom of the spine at the L4-5 or L5-S1 segments. This T1-weighted MRI reveals posterolateral herniation of the nucleus pulposus of the L5-S1 intervertebral disc, as indicated by the white arrow. Remember that a herniated nucleus pulposus may displace or disrupt the posterior longitudinal ligament and extend into the vertebral canal and impinge upon spinal nerve roots either within the vertebral canal or as they traverse the intervertebral foramen (or both). When dealing with such cases in the lower lumbar regions, remember the formula "N+1." That is, N = the number of the intervertebral disc; +1 = the number of the spinal nerve roots primarily contacted by the herniation. Thus, the herniated L5-S1 intervertebral disc would impinge the S1 nerve roots, which resulted in the lower back pain, tingling numbness radiating down his right lower limb to the lateral part of his foot, and weakness when standing on the toes of his right foot.

After moving his parents' heavy furniture out of their house, a 38-year-old man experiences lower back pain and presents to his doctor with tingling numbness radiating down his right lower limb to the lateral part of his foot. A thorough physical examination reveals weakness when standing on the toes of his right foot. A T1-weighted MRI revealed an intervertebral disc herniation, as indicated by the white arrow. Which of the following structures is most likely impinged by this disc herniation? (A) S1 spinal nerve roots (B) S1 anterior primary ramus (C) L5 spinal nerve roots (D) L5 posterior primary ramus (E) L5 anterior primary ramus

*The answer is C.* Classic posterolateral herniation of the nucleus pulposus of the intervertebral disc occurs most often within the C4-5 and L4-5 intervertebral discs. A herniated nucleus pulposus may displace or disrupt the posterior longitudinal ligament and extend into the vertebral canal and impinge upon spinal nerve roots either within the vertebral canal or as they traverse the intervertebral foramen (or both). When dealing with such cases in the cervical and lower lumbar regions, remember the formula "N+1." That is, N = the number of the intervertebral disc; +1 = the number of the spinal nerve roots primarily contacted by the herniation. Thus, the herniated C5-6 disc will most likely impinge upon the C6 spinal nerve roots. In the cervical region, this formula works because there are eight pairs of cervical spinal nerves but only seven cervical vertebrae. Each numbered spinal nerve exits above the matching numbered vertebra. Thus, the C6 cervical spinal nerve exits the intervertebral foramen formed by the C5 and C6 vertebrae, where it may be impinged. The formula also works in the low lumbar region because of the acute angles the nerve roots take in entering the intervertebral foramina. Here, the numbered spinal nerves exit below the matching vertebrae, so the L4 nerve roots enter the intervertebral foramen between the L4 and L5 vertebrae. Because these lumbar nerve roots take a very acute turn to enter that opening, they are typically held against the upper pedicle and are superior to the level of the intervertebral disc. Thus, the L4 roots would not be impinged by a bulging L4-5 disc. Therefore, the L4-5 intervertebral disc would, instead, push primarily against the L5 roots as they align to enter the intervertebral foramen formed by the L5 and S1 vertebrae.

After several months of engaging in a vigorous exercise program, a 28-year-old woman experiences periodic pain and muscle spasms in her left upper limb. Thorough physical and radiographic examinations by her primary care physician and a consulting surgeon determine that she suffers a posterolateral herniation of the C5-6 intervertebral disc. Which of the following structures is this herniation most likely impinging? (A) C5 spinal nerve roots (B) C5 anterior primary ramus (C) C6 spinal nerve roots (D) C6 posterior primary ramus (E) C7 posterior horn segment of the spinal cord

*The answer is A.* Cell bodies of somatic motor neurons (a-motor neurons) innervating skeletal muscle are located within the anterior (ventral) gray horn of the spinal cord, at all segmental levels throughout the entire length of the spinal cord. The innervation of the skeletal muscles affected by ALS is through somatic motor (general somatic efferent or GSE) neurons and branchial motor (special visceral efferent or SVE) neurons (neurons that supply the embryonic pharyngeal arches). In ALS patients, postmortem analysis of the anterior gray horn of the spinal cord would show significant degeneration.

Amyotrophic lateral sclerosis (ALS; Lou Gehrig's disease) is a progressive, fatal neurodegenerative disease caused by degeneration of the motor neurons controlling skeletal (voluntary) muscle movement. Postmortem analysis of which of the following structures would show the cell bodies of neurons affected by this disease? (A) Anterior gray horn of the spinal cord (B) Lateral gray horn of the spinal cord (C) Posterior gray horn of the spinal cord (D) Spinal ganglia (E) Lateral column of spinal cord white matter

*The answer is C.* The photo demonstrates a case of "winged scapula," indicative of lesion of the long thoracic nerve and subsequent paralysis of the serratus anterior muscle. The nerve is located on the lateral thoracic wall, on the superficial aspect of the serratus anterior, where it is not afforded the protection of the muscle it innervates (like most motor nerves), especially when the limb is elevated. In this patient, shrapnel wounds to the lateral thoracic wall caused damage to the long thoracic nerve and subsequent loss of innervation to the serratus anterior muscle. When the affected limb is protracted, the medial border and inferior angle of the scapula pull away from the posterior chest wall, giving the scapula a wing-like appearance. Additionally, the affected arm cannot be abducted above the horizontal plane because the serratus anterior is not available to superiorly rotate the glenoid cavity of the scapula to allow full abduction. Following damage to the long thoracic nerve, it takes several weeks for a winged scapula to the develop because the trapezius muscle, which attaches to the spine of the scapula, must stretch before winging is apparent.

An 18-year-old soldier presents with shrapnel wounds in the lateral wall of his right chest following an explosion of a landmine. After several months of recovery, his physical therapist observes that his scapula moves away from the thoracic wall when he leans on his right hand, as noted by the black arrow in the given photo. Which of the following nerves is likely damaged? (A) Axillary nerve (B) Thoracodorsal nerve (C) Long thoracic nerve (D) Dorsal scapular nerve (E) Suboccipital nerve

*The answer is B.* The iliocostalis thoracis muscle is found in the deep back and functions to maintain posture. It is not associated with neck flexion. The sternocleido-mastoid muscle is innervated by CN XI and functions in contralateral rotation (unilateral contraction) and flexion (bilateral contraction) of the neck. Rhomboid major and minor are both innervated by the dorsal scapular nerve and serve to adduct the scapulae. Teres major is innervated by the lower subscapular nerve and serves to medially rotate and adduct the humerus.

An 18-year-old woman passenger injured in a rollover car crash was rushed to the emergency department. After the patient is stabilized, she undergoes physical examination. She demonstrates considerable weakness in her ability to flex her neck, associated with injury to CN XI. Which of the following muscles is most probably affected by nerve trauma? A. Iliocostalis thoracis B. Sternocleidomastoid C. Rhomboid major D. Rhomboid minor E. Teres major

*The answer is B.* The posterior (dorsal) and anterior (ventral) roots of spinal nerves typically merge within the intervertebral foramina to form individual spinal nerves. Because the spinal (dorsal root) ganglia are located at the distal ends of the posterior roots, they are also normally found within the intervertebral foramina. Thus, narrowing (stenosis) or compression of the intervertebral foramina may cause impingement of the posterior and anterior roots, spinal nerves, and spinal ganglia due to the presence of bony spurs (osteophytes) in this patient.

An 81-year-old woman visits her family physician for an annual physical examination and complains of weakness and loss of sensation in her legs. Radiological studies show bony spurs (osteophytes) narrowing the intervertebral foramina at levels T12-L3. Which of the following structures is most likely to be impinged in this condition? (A) Posterior primary rami of spinal nerves (B) Spinal ganglia (C) Ganglia of the sympathetic trunk (D) Anterior primary rami of spinal nerves (E) Anterior rootlets of spinal nerves

*The answer is A.* The ligamentum flavum (L: yellow ligament) consists of yellow elastic fi brous tissue, which binds together the laminae of adjoining vertebrae and forms the posterior wall of the vertebral canal. During proper administration of an epidural anesthetic, the needle will pass (in order) through the supraspinous ligament overlying the spinous processes of the lumbar vertebrae, the interspinous ligament connecting the spinous processes of adjacent vertebrae, and fi nally the ligamentum fl avum, which stretches between the laminae of adjacent vertebrae. Due to its high elastic fiber content and the usual placement of the pregnant woman into the fetal position, the anesthesiologist will feel substantial resistance before the needle passes through the ligamentum flavum and potentially an audible "pop" when it is penetrated. In the given diagram, the epidural space resides between the ligamentum flavum and the posterior longitudinal ligament. Remember that the ligamentum flavum is the last ligament the needle penetrates during administration of an epidural anesthetic.

An anesthesiologist administers epidural anesthestic immediately lateral to the spinous processes of vertebrae L3 and L4 of a pregnant woman in labor. During this procedure, what would be the last ligament perforated by the needle in order to access the epidural space? (A) Ligamentum flavum (B) Anterior longitudinal ligament (C) Posterior longitudinal ligament (D) Interspinous ligament (E) Intertransverse ligament

*The answer is A.* The order of structures pierced during an epidural procedure is skin, subcutaneous tissue, muscle, supraspinous ligament, interspinous ligament, and ligamentum flavum (there is often a midline gap in the ligamentum flavum). The anterior longitudinal ligament is anterior to the vertebral body and cannot be reached by this approach. The posterior longitudinal ligament is posterior to the vertebral body and can also not be reached by this procedure. The intertransverse ligaments are too lateral and may not be perforated by this technique.

An anesthesiologist administers epidural anesthetic immediately lateral to the spinous processes of vertebrae L3 and L4 of a pregnant woman in labor. During this procedure, what would be the last ligament perforated by the needle in order to access the epidural space? A. Ligamentum flavum B. Anterior longitudinal ligament C. Posterior longitudinal ligament D. Interspinous ligament E. Intertransverse ligament

*The answer is D.* The cerebrospinal fluid (CSF) is located in the subarachnoid space, between the arachnoid layer and pia mater. In a lumbar puncture, the needle penetrates the skin, fascia, ligamentum flavum, epidural space, dura mater, subdural space, and arachnoid mater. The pia mater forms the internal boundary of the subarachnoid space; thus, it cannot be penetrated by needle. The posterior longitudinal ligament lies anterior to the spinal cord; thus, it is not penetrated by the needle. The filum terminate externum is the downward prolongation of the spinal dura mater from the second sacral vertebra to the dorsum of the coccyx. The anulus fibrosus consists of concentric layers of fibrous tissue and fibrocartilage surrounding and retaining the nucleus pulposus of the intervertebral disk, which lies anterior to the spinal cord.

An elderly man at a nursing home is known to have degenerative brain disease. When cerebrospinal fluid (CSF) is withdrawn by lumbar puncture for further examination, which of the following structures is most likely penetrated by the needle? (A) Pia mater (B) Filum terminale externum (C) Posterior longitudinal ligament (D) Ligamentum flavum (E) Anulus fibrosus

*The answer is B.* The triangle of auscultation is a space in the back bounded by the lateral edge of the trapezius muscle, medial border of the scapula, and upper edge of the latissimus dorsi muscle. Placing a stethoscope within the triangle of auscultation enables the medical student to hear breathing sounds because (1) this site offers a gap between layers of bone and muscle, and (2) the location lies directly over the midposterior chest wall and the lung, which is ideal for auscultation.

An internal medicine attending physician asks a medical student to place the bell of her stethoscope on the triangle of auscultation to hear a patient's breathing sounds. Which of the following structures make up the boundaries of this triangle? (A) Deltoid, latissimus dorsi, scapula (B) Trapezius, scapula, latissimus dorsi (C) Latissimus dorsi, ilium, external abdominal oblique (D) Rhomboids, levator scapulae, splenius capitis (E) Longissimus, rhomboids, vertebral spinous processes

*The answer is B.* The sagittal CT clearly shows a Type II dens axis (odontoid process) fracture, which is the most common type of fracture (approximately 60% of cases) involving the dens. This fracture occurs at the base of the dens where it extends superiorly from the axis (or second cervical vertebra). The precise mechanism for odontoid fractures is unknown, but it most likely includes a combination of flexion, extension, and rotation. In this case, the patient's cervical spine was forced into a flexed position by the impact of the crash and then a hyperextended position with slight rotation due to the impact of her head on the steering wheel. The forces involved fractured the odontoid process at its base, but luckily, the fractured dens axis does not extend posterior to impinge upon the spinal cord. Patients with this type of odontoid fracture are placed in halo immobilization or undergo internal fixation (or odontoid screw fixation) to reattach the fractured axis dens to the body of the axis. Remember Steele's Rule of Thirds, which states one third of the atlas (C1) ring is occupied by the dens axis, one third by the spinal cord, and one third by the fluid-fi lled space and surrounding tissues of the cord. This extra space within the atlas ring explains why people with odontoid fractures may not have spinal cord injuries, but they will often feel unstable when they move their head suddenly. The incidence of odontoid fractures approaches 15% of all cervical spine fractures.

During a head-on motor vehicular accident (MVA), the upper cervical vertebrae of a 34-year-old woman are flexed violently forward until her head impacts the steering wheel and is thrown into hyperextension and slight rotation. She is immobilized and brought to an ER. The given sagittal CT of her upper cervical vertebrae is viewed by a radiologist and reveals which of the following diagnoses? (A) Traumatic spondylolisthesis of C2 (B) Fracture of dens axis (odontoid process) (C) Atlanto-axial subluxation (D) Ruptured intervertebral disc between C1 and C2 (E) No pathology is apparent on the CT scan

*The answer is D.* Cerebrospinal fluid (CSF) is found in the subarachnoid space, which is a wide interval between the arachnoid layer and the pia mater. The epidural space contains the internal vertebral venous plexus and epidural fat. The subdural space between the arachnoid and the dura contains a little fluid to moisten the meningeal surface. The pia mater closely covers the spinal cord and en meshes blood vessels on the surfaces of the spinal cord. Thus, the space between the spinal cord and the pia is a potential space.

During an outbreak of meningitis at a local college, a 20-year-old young student presents to a hospital emergency room complaining of headache, fever, chills, and stiff neck. On examination, it appears that he may have meningitis and needs a lumbar puncture or a spinal tap. Cerebrospinal fluid (CSF) is normally withdrawn from which of the following spaces? (A) Epidural space (B) Subdural space (C) Space between the spinal cord and pia mater (D) Subarachnoid space (E) Space between the arachnoid and dura maters

*The answer is A.* The anterior longitudinal ligament is a vertical connective tissue band that attaches along the anterior aspects of the vertebral bodies. Its peripheral fibers have strong attachments to the intervertebral discs. The anterior longitudinal ligament resists hyperextension of the vertebral column. However, in this patient, the extreme forces involved with the hyperextension of the neck overpowers the resistance of this ligament, rupturing it as well as displacing part of the C4-5 intervertebral disc. In the given T2-weighted MRI, the anterior longitudinal ligament is represented by a hypointense (dark band) signal located anterior to the vertebral column. However, the locations where the anterior longitudinal ligament is interrupted appear as an abnormal hyperintense (white) signal, which is evident anterior to the C5 vertebral body.

Idling at a stoplight in his vintage car without headrests, a 71-year-old-man's car is struck from behind by a truck going approximately 30 mph (48 kph). The man is brought to the ER suffering from a severe hyperextension neck injury due to the crash. The given T2-weighted MRI shows a rupture of the anterior anulus of the C4-5 intervertebral disc, inflammation of that disc (the white appearance), and a prevertebral hematoma, which compromised his airway and required intubation. Which of the following ligaments is disrupted in this injury? (A) Anterior longitudinal ligament (B) Posterior longitudinal ligament (C) Ligamentum fl avum (D) Interspinous ligament (E) Intertransverse ligament

*The answer is C.* There are seven (7) cervical vertebrae and eight (8) cervical spinal nerves. Nerves C1 to C7 exit superior to their corresponding vertebrae, whereas nerve C8 exits inferiorly to the C7 vertebra. The nerves of the thoracic and subsequent regions all exit inferior to their corresponding vertebrae.

In a report of a radiograph of the cervical spine the radiologist wrote the following: "Severe narrowing of the C7-T1 intervertebral foramen (IVF) on the left." Which nerve was most likely compressed as a result of this finding? A. C6 B. C7 C. C8 E. T1

*The answer is C.* This is the location of the conus medullaris, a tapered conical projection of the spinal cord at its inferior termination. Although the conus medullaris rests at the level of L1 and L2 in adults, it is often situated at L3 in newborns. The cauda equina and filum terminale extend beyond the conus medullaris.

In spinal anesthesia, the needle is often inserted between the spinous processes of the L4 and L5 vertebrae to ensure that the spinal cord is not injured. This level is safe because in the adult the spinal cord usually terminates at the disc between which of the following vertebral levels? A. T11 and T12 B. T12 and L1 C. L1 and L2 D. L2 and L3 E. L3 and L4


Ensembles d'études connexes

Biomatek összes tesztkérdés 2022 - válaszlehetőségekkel

View Set

gov CH 6: Interest Groups and Lobbying

View Set

Exam 4: International Business (Ch 16)

View Set

Chapter 9 Teaching and Counseling

View Set

Saunders: Cancer & Gender Specific Cancer -- breast, ovarian, cervical, and prostate

View Set